Which of the Following is Next Best Management?

Question:

A 48-year-old woman presents with progressive muscle weakness, primarily in the proximal muscles of her upper and lower extremities, along with occasional shortness of breath. Physical examination reveals a rash over her knuckles and a heliotrope rash around her eyes. Laboratory testing shows elevated levels of creatine kinase (CK) and lactate dehydrogenase (LDH). Electromyography (EMG) reveals abnormal spontaneous muscle activity, and muscle biopsy shows perifascicular atrophy. Further testing reveals positive anti-Jo-1 antibodies. Given the patient’s clinical findings and diagnosis, which of the following is the next best step in management?

A) High-dose corticosteroids with gradual tapering
B) Azathioprine for long-term immunosuppression
C) Pulmonary function tests (PFTs)
D) Intravenous immunoglobulin (IVIG) therapy
E) Methotrexate for maintenance therapy

Correct Answer: C) Pulmonary function tests (PFTs)

Immune Trigger (Anti-Jo-1 antibodies) 🧠
⬇️
Activation of Autoimmune Response

⬇️
T-cells and cytokines ↑ target muscle and skin cells

⬇️
 Complement System Activation (specifically MAC - Membrane Attack Complex)

⬇️
Complement deposits in capillaries cause microvascular damage

⬇️
Ischemia in affected muscle fibers (especially perifascicular)

⬇️

Capillary Injury in Muscle and Skin 🧠
⬇️


Hypoxia ⬇️ oxygen to muscle fibers near capillaries

⬇️

Muscle Fiber Degeneration

⬇️
Perifascicular Atrophy seen on biopsy 🧠

⬇️
Release of muscle enzymes (↑ CK, LDH due to muscle damage)
⬇️

Muscle Symptoms
Proximal Muscle Weakness (affects shoulders, hips)
Difficulty with tasks: lifting arms, climbing stairs

⬇️
Iimmune-mediated damage in skin capillaries

Skin Involvement
 Gottron’s Papules (red-purple papules on knuckles)
Heliotrope Rash (purple discoloration around eyes)
⬇️
Immune System Targets Lung Tissue

⬇️
Chronic inflammation in lung interstitium (alveoli and interstitial spaces)

⬇️


Fibrosis (scarring) forms as a response to inflammation


⬇️

Pulmonary Symptoms (ILD)
🔵 Shortness of breath
🔵Decreased gas exchange due to fibrosis

⬇️

Needs Pulmonary Function Tests (PFTs) to assess lung function 🧠

Why Other Choices are Incorrect


A (High-dose corticosteroids): Needed to manage inflammation but secondary to assessing ILD risk with PFTs.


B (Azathioprine) & E (Methotrexate): Immunosuppressants useful for long-term control but not prior to ILD assessment.


D (IVIG therapy): Can benefit severe, refractory cases, but confirming lung involvement with PFTs is essential first.

Stay Hungry, Stay Curious !

Your Brother In This Struggle

Dr. Shoaib Ahmad